Using the figure below as a starting point, identify the figure in which lines to l are drawn through points A, B, C, and D.

Using The Figure Below As A Starting Point, Identify The Figure In Which Lines To L Are Drawn Through

Answers

Answer 1

SOLUTION

We want to find the figure in which lines perpendicular to l are drawn through points A, B, C, and D

The correct figure will be the one in which a vertical line is drawn across each of points A, B, C and D.

Looking at this, we can see that the correct answer is the first option

Answer 2

Answer:

a

Step-by-step explanation:


Related Questions

2) Given that XY || AC, what is YC if BX = 10, BA = 15, and BY = 8?A) 4 B) 6 C)8D)12

Answers

We can see that triangles ABC and AXY are congruent

This means that

[tex]\frac{AX}{BX}=\frac{YC}{BY}[/tex]

Now, we know that BX=10, BY=8 and BA=AX+BX, hence AX=BA-BX, we have

[tex]\frac{BA-BX}{10}=\frac{YC}{8}[/tex]

now, since BA-BX=15-10, BA-BX=5, it yields,

[tex]\frac{5}{10}=\frac{YC}{8}[/tex]

Now, we need to isolate YC, this is given by

[tex]YC=8(\frac{5}{10})[/tex]

Since

[tex]\frac{5}{10}=\frac{5\cdot1}{5\cdot2}=\frac{1}{2}[/tex]

we have that

[tex]\begin{gathered} YC=8(\frac{1}{2}) \\ YC=\frac{8}{2} \\ YC=4 \end{gathered}[/tex]

hence, the answer is YC=4, which corresponds to A).

Use the following function for questions # 1 - # 5:f(x) =x?- 14x + 44#1: Find the X value of the turning point.

Answers

The given function is

f(x) = x^2 - 14x + 44

To find the turning point, we would differentiate the function, equate the derivative to zero and solve for x. We have

f'(x) = 2x - 14

Equating it to zero, we have

2x - 14 = 0

2x = 14

x = 14/2

x = 7

The value of x of the turnng point is 7

According to the graph, what is the value of the constant in the equation below? 2- 18+ Height = Constant Wiat 16+ (0.5,1.6) 12+ Height (0.8.1) 0.8 + (1.6.05) (2.0.4) 12 14 16 18 2

Answers

Liyah, this is the solution:

• Height = Constant/Width

,

• Height * Width = Constant (you need to multiply each ordered pair )

Therefore,

Constant = 1.6 * 0.5

Constant = 0.8

Constant = 0.4 * 2

Constant = 0.8

The correct answer is C. 0.8

What is an equation of the line parallel to the line on the graph that passes through (2,25)?

Answers

y=4x+17

Explanation

Step 1

2 equations of lines are parallel if the slope is the same, so

a) find the slope of the graphed line

the slope of a line can by calculated by using

[tex]\begin{gathered} slope=\frac{change\text{ in y }}{change\text{ in x}}=\frac{y_2-y_1}{x_2-x_1} \\ where \\ P1(x_1,y_1) \\ and \\ P2(x_2,y_2) \\ are\text{ 2 points from the line} \end{gathered}[/tex]

so

pick up 2 points from the the line and let

[tex]\begin{gathered} P1(0,10) \\ P2(10,50) \end{gathered}[/tex]

replace and evaluate

[tex]\begin{gathered} slope=\frac{change\text{\imaginaryI ny}}{change\text{\imaginaryI nx}}=\frac{y_{2}-y_{1}}{x_{2}-x_{1}} \\ slope=\frac{50-10}{10-0}=\frac{40}{10}=4 \end{gathered}[/tex]

hence, the slope of the line is 4

Step 2

now, using the slope and a point we can find the equation of the line

use the point-slope formula, it says

[tex]\begin{gathered} y-y_1=m(x-x_1) \\ where\text{ m is the slope} \\ (x_1,y_1)\text{ is a point from the line} \end{gathered}[/tex]

so

a)let

[tex]\begin{gathered} P1(2,25) \\ sloipe=4 \end{gathered}[/tex]

b) now ,replace and solve for y

[tex]\begin{gathered} y-y_{1}=m(x-x_{1}) \\ y-25=4(x-2) \\ y-25=4x-8 \\ add\text{ 25 in both sides} \\ y-25+25=4x-8+25 \\ y=4x+17 \end{gathered}[/tex]

so, the answer is

y=4x+17

I

Identify the domain and range for the given relation. Indicate whether the relation is a function or not andexplain

Answers

Given :

Domain is:

[tex]D\colon\mleft\lbrace0,-1,1\mright\rbrace[/tex]

Range is:

[tex]R\colon\mleft\lbrace0,1,2\mright\rbrace[/tex]

Here is one output for one input.

Suppose that the functions fand g are defined for all real numbers x as follows.f(x)=x+5g(x)=2x²Write the expressions for (g+f)(x) and (g–f)(x) and evaluate (g.f)(-3).

Answers

The expression (g+f)(x) is equal to g(x)+ f(x), (g-f)(x) is equal to g(x) f(x) and the expression (g*f)(-3) is equal to g(-3)*f(-3).

Then, we have

[tex](g+f)(x)=g(x)+f(x)=2x^2+x+5[/tex]

Similarly,

[tex](g-f)(x)=g(x)f(x)=2x^2-(x+5)=2x^2-x-5[/tex]

And finally,

[tex]\begin{gathered} (g\cdot f)(-3)=g(-3)\cdot f(-3)=2(-3)^2\cdot(-3+5) \\ (g\cdot f)(-3)=2(9)\cdot(2) \\ (g\cdot f)(-3)=36 \end{gathered}[/tex]

In summary, the answers are:

[tex]\begin{gathered} (g+f)(x)=2x^2+x+5 \\ (g-f)(x)=2x^2-x-5 \\ (g\cdot f)(-3)=36 \end{gathered}[/tex]

The number of adults living in homes on a randomly selected city block is described by the following probability distribution. Number of adults, x1 ,2,3,4 or moreProbability, P(x) 0.250.500.15??? What is the probability that 4 or more adults reside at a randomly selected home?(A) 0.10(B) 0.15 (C) 0.25(D) 0.50 (E) 0.90

Answers

The answer is letter A. 0.1

because the probability = 1

So Probability of select 4 adults = 1 - 0.25 - 0.5 - 0.15

= 0.1

Y 3+ 2+ 1+ -4 -3 -2 -1 1 2 3 -1- -2 -3+ -4 47 What is the slope of the line?

Answers

To find the slope of the line, we will follow the steps given below:

Step 1: select two points on the graph

(0, -1) and (4,2)

Step 2: Apply the slope formula:

[tex]\text{slope}=\frac{y_2-y_1}{x_2-x_1}[/tex]

=>

[tex]\text{slope}=\frac{2-(-1)}{4-0}=\frac{2+1}{4}=\frac{3}{4}[/tex]

The slope of the graph is:

[tex]\frac{3}{4}[/tex]

What is the measure of

Answers

In the parallelogram ABCD,

Angle D is 145 degree.

In the parallelogram adjacent angles sum is 180 degree.

In the given parallelogram ABCD , angle D and angle C are adjacent.

[tex]\begin{gathered} \angle D+\angle C=180^{\circ} \\ 145^{\circ}+\angle C=180^{\circ} \\ \angle C=180^{\circ}-145^{\circ} \\ \angle C=35^{\circ} \end{gathered}[/tex]

Answer: Option B) 35 degree.

An empty swimming pool needs to be filled to the top. The pool is shaped like a cylinder with a diameter of 9 m and a depth of 1.1 m. Suppose water is pumped into the pool at a rate of 13 m^3 per hour. How many hours will it take to fill the empty pool?

Use the value 3.14 for pi, and round your answer to the nearest hour. Do not round any intermediate computations.

Answers

Answer:

πr2h

volume of cylinder

3.14×3×3×1.1=31.086m^3

1hour=13m^3

31.086m^3

divide 31.086÷13=2.3912hours

Which number is not equal to 225%?its is exercise number 5

Answers

To do this, you can first convert the percentage form to its decimal form, like this

[tex]225\text{\%}=\frac{225}{100}=2.25[/tex]

Now, you can convert the numbers that are possible answers into their decimal form, like this

Option A.

[tex]2\frac{1}{4}=\frac{2\cdot8+1}{4}=\frac{8+1}{4}=\frac{9}{4}=2.25[/tex]

Option B.

[tex]\frac{9}{4}=2.25[/tex]

Option C.

[tex]\frac{50}{40}=\frac{5\cdot10}{4\cdot10}=\frac{5}{4}=1.25[/tex]

Option D.

[tex]\frac{45}{20}=\frac{9\cdot5}{4\cdot5}=\frac{9}{4}=2.25[/tex]

Therefore, the number that is not equal to 225% is 50/40 and the correct answer is C. 50/40.

help me asap please on this math question

Answers

Equations showing direct variations are 2x = y and y = 1.8c

Direct Variation exists between two variables when one variable is directly dependent to another variable means change in one variable will create change in other one also and vice versa.

Two variable increase or decrease by the same factor.

Suppose x and y is that are in direct variation then you can write

y ∝ x

where, "∝" denotes proportionality

removing proportionality sign by constant then you can write

y = k x , where k is constant and can hold any real value

From the following equation ,

2x = y with 2 as constant and

y = 1.8x with 1.8 as constant shows direct variations

To know more about Direct variation here

https://brainly.com/question/18087321

#SPJ1

(a) Does f (x) have a horizontal asymptote? If so, what is it?(b) Does f (x) have any vertical asymptotes? If so, what are they?

Answers

a) Horizontal asymptotes are horizontal lines that the graph of a function approaches but never touches. To find the horizontal asymptote, we would apply one of the rules which states that

If the degree of the of the denominator is bigger than the degree of the numerator, the horizontal asymptote is the x axis of the graph. It occurs at y = 0

The degree is the largest exponent in the function. Looking at the given function, the degree of the numerator is 2 while the degree of the denominator is 3. Thus,

there is a horizontal asymptote at y = 0

b) The vertical asymptotes are vertical lines which correspond to the zeros of the denominator of rational functions. It is equal to the values of x that make the denominator to be zero. Looking at the given function, (x + 1) cancels out in the numerator and denominator. We are left with (x - 4) and (x + 5). We would equate both terms to zero and solve for x. These values of x would make the denominator to be equal to zero. We have

x - 4 = 0

x = 4

x + 5 = 0

x = - 5

Thus,

there are vertical asymptotes at x = - 5 and x = 4

What is the surface area of the regular pyramid below?A. 648 sq. unitsB. 552 sq. unitsC. 396 sq. unitsD. 522 sq. units

Answers

Step 1:

Concept: Calculate the area of each face and add all together to get the surface area of the pyramid.

The regular pyramid below have 4 triangles and a square

Step 2: Apply the area formula to find the area of the 4 triangles and a square.

[tex]\begin{gathered} \text{Area of a triangle = }\frac{Base\text{ x Height}}{2} \\ \text{Area of the square base = Length x Length} \end{gathered}[/tex]

Step 3:

Given data for the triangle

Height = 21

Base = 12

[tex]\begin{gathered} Area\text{ of a triangle = }\frac{Base\text{ x Height}}{2} \\ =\text{ }\frac{21\text{ x 12}}{2} \\ =\text{ }\frac{252}{2} \\ =126\text{ sq. units} \\ \text{Area of the four triangles = 4 x 126 = 504 sq. units} \end{gathered}[/tex]

Step 4: Find the area of the square

Given data for the square

Length = 12

Area = length x length = 12 x 12 = 144 sq. units

Step 5: Add the area of the four triangles and the square.

Surface area of the regular pyramid = 504 + 144

= 648 sq. units

5Jamal's band learns lots of new songs.The band learns a new song every fourdays. At this rate, how many new songswill the band learn in four weeks?LAsongs

Answers

Given that The band learns a new song every four days.

We need to find the number of new songs for four weeks.

We know that one week= 7 days

[tex]1\text{ w}eek\text{ = 7 days }[/tex]

Multiply by 4 to find the number of days in four weeks.

[tex]1\times4\text{ w}eek\text{ = 7 }\times4\text{ days }[/tex]

[tex]4\text{ w}eeks\text{= 28days }[/tex]

We need to find the number of new songs that the band learns in 28 days.

Divide 28 by 4 to find the number of songs since the band learns one new song every 4 days.

[tex]\frac{28}{4}=7\text{ songs}[/tex]

The band learns 7 songs in four weeks.

3 2 1 -3-2- 1 2 3 2 -3 Domain: (-3,3] Range: [-2, 2] Domain: (-2, 2] Range: [-3,3] Domain: (-2,-3) Range: (2,3) Domain: {-2, -1, 0, 1, 2} Range: {-3, -2, - 1, 0, 1, 2, 3} None of the above NON

Answers

The domain is [ -2, 2]

and the range is [-3, 3]

find the smallest non negative value for x in degrees that makes the equation cot (x) = √3 true.

Answers

Given:

cot (x) = √3

To find the smallest non-negative value for x in degrees:

So, we get

[tex]\begin{gathered} \cot x=\sqrt{3} \\ \cot x=\cot (30^{\circ}) \\ x=30^{\circ} \end{gathered}[/tex]

Hence, the answer is,

[tex]x=30^{\circ}[/tex]

A __ is a polynomial with one term.

Answers

Answer:

Monomial

Step-by-step explanation:

A polynomial that consists of exactly one term is called monomial.

Examples are 3, 10x², xy,...

So the answer is: Monomial

The price of a train ticket consists of an initial fee of $5 plus a fee of $2.75 per stop. Julia has $21 and would like to travel 50 kilometers. She wants to know the largest number of stops she can afford to buy on a ticketLet S represent the number of stops that Julia buys.1) Which inequality describes this scenario?A. 5+2.75•S ≤ 21 B. 5+2.75•S ≥ 21 C. 5+2.75•S ≤ 50 D. 5+2.75•S ≥ 502) What is the largest number of stops that Julia can afford?

Answers

Let's begin by listing out the information given to us:

Initial fee = $5

Fee per stop = $2.75

Amount with Julia = $21

What is the highest number of stops she can make?

S = the number of stops Julia bought

Julia pays the initial fee of $5. We subtract this from the $21, we have

$ (21 - 5) = $16

Julia has $16 left to buy her stops. She cannot spend beyond the amount of money with her (altogether $21). She spends lesser than or equal to $21 (≤ $21)

The inequality that describes this scenario is given by:

initial fee + fee per stop * number of stops ≤ 21

5 + 2.75 * S ≤ 21

Hence, option A is the correct answer

What is the largest number of stops that Julia can afford?

This is gotten by dividing the amount left after subtracting the initial fee by the fee per stop

n = 16/2.75 = 5.82 = 5 stops (rounding downwards)

We round downwards because the number of stops must be a whole number and it must be lesser than or equal to $21 altogether

9. At last Friday's soccer game there were a total of 673 fans in attendance, including students and non-students.Let x represent the number of students, and y represent non-students. Which of the following statements couldrepresent the number of fans in attendance. Select all that apply.a. x + y = 673b. 335 and 138c. 335 and 338d. x=y - 673e. y = -x + 673f. 273 and 400

Answers

The answer is A

From the question:

Total fans in attendence = 673

x = number of students

y= non - students

Total of fans in attendance =

x + y= 673

Consider the following simple statements: p: Your shirt is tucked into your pants. q: Your pants are tucked into your shirt.What is the symbolic form of the statement: "If your shirt isn't tucked into your pants then your pants are tucked into your shirt."Select the correct answer below:∼q⟹p∼q⟹∼pp⟹∼q∼p⟹q

Answers

SOLUTION

We are asked the symbolic form of "If your shirt isn't tucked into your pants then your pants are tucked into your shirt."

This simply means the negation of p implies q.

p implies q is represented as p⟹q

Then the negation of p implies q will be ∼p⟹q.

Therefore, the correct answer is ∼p⟹q

hi I need help ;]]] ‍❄️‍❄️‍❄️

Answers

The order from least to greatest is -4.7,-4,-31/8, [tex]-3\frac{1}{8}[/tex]

What is Fraction?

A fraction represents a part of a whole.

The given integers are -31/8,-4.7, -4 and -3 1/8

Now let us simplify the fraction values

-31/8=-3.875

-4.7

-4 and

     [tex]-3\frac{1}{8}[/tex]=-24+1/8=-23/8=-2.875

As there is a negative sign, the smallest number with negative sign will be greatest and largest number with negative sign is smaller.

So -4.7,-4, -3.875, -2.875

-4.7,-4,-31/8, [tex]-3\frac{1}{8}[/tex]

Hence the order from least to greatest is -4.7,-4,-31/8, [tex]-3\frac{1}{8}[/tex]

To learn more on Fractions click:

https://brainly.com/question/10354322

#SPJ1

8) Suppose y varies inversely as x, if y = 7 when x = 6then find y when x = -21.

Answers

Suppose y varies inversely as x, if y = 7 when x = 6

then find y when x = -21.

we know that

A relationship between two variables, x, and y, represent an inverse variation if it can be expressed in the form

y*x=k

where

k is the constant of proportionality

step 1

Find the value of k

we have

y=7, x=6

k=7*6

k=42

the equation is

y*x=42

step 2

For x=-21

substitute

y*(-21)=42

y=42/(-21)

y=-2

12x+18 rewrite using distributive property

Answers

we have

12x+18

REmember that

12=(2^2)*(3)

18=(2)*(3^2)

substitute

(2^2)*(3)x+(2)*(3^2)

Factor (2)*(3)=6

6(2x+3)

therefore

the answer is

6(2x+3)

How many gallons are equivalent to 12 quarts?

Answers

Given:

The objective is to convert 12 quarts into gallons.

In general,

[tex]1\text{ gallon=4}quart[/tex]

12 quarts can be converted to gallons by,

[tex]\begin{gathered} x=\frac{12}{4} \\ x=3\text{ quarts} \end{gathered}[/tex]

Hence, 12 quarts is 3 gallons.

3 gallons !! Hope this helped

the radius of a circle is 4 centimeters. what is the diameter? give the exact answer in simplest form

Answers

we have that

the diameter is two times the radius

so

in this problem

D=2r

D=2(4)=8 cm

diameter is 8 cm

help me asap please!!! no explanation just the process and answer

Answers

To find out the determinant, multiply in cross

so

(3)*(-2)-(5)*(-7)=-6+35=29

therefore

the answer is 29

Simplify the expression cos x/ cot x.a. cos xb. tan xc. sin xd. cos²x/sin x

Answers

cosx/cotx = cosx *tan x =cosx (sinx/cosx) = sin x

Answer

c. sin x

What is 132% as a decimal?

Answers

Step 1: Problem

What is 132% as a decimal?​

.132 is your answer because if you where getting the percent off of something you would do that number X divided by .132 to get the answer.

You flip a coin and roll a die. The table shows the sample space.12 3 4 5 6Heads(H) H-1 H-2 H-3 H-4H-5H-6Tails(T) T-1 T-2 T-3 T-4 T-5 T-6What is the probability of getting a head or a tail and anven number?Answer as a reduced fraction in the form ab.

Answers

You flip a coin and roll a die. The table shows the sample space.

1

2 3 4 5 6

Heads(H) H-1 H-2 H-3 H-4H-5H-6

Tails(T) T-1 T-2 T-3 T-4 T-5 T-6

What is the probability of getting a head or a tail and an

even number?

we know that

The probability of an event is the ratio of the size of the event space to the size of the sample space.

The size of the sample space is the total number of possible outcomes

The event space is the number of outcomes in the event you are interested in.

In this problem

the size of the sample space is (6+6+6)=18

the size of the event space is equal to (6+6+3)=15

REmember that an even number are (2,4 and 6)

so

the probability is equal to

P=15/18

simplify

P=5/6

therefore

the answer is5/6

Other Questions
What causes water to maintainits temperature whether it ishot or cold?A. Water is a very large molecule.B. Water has a high heat capacity. ThisC. Water has very strong covalent bonds.D. Water is a non-polar molecule. In a class of 30 students, 3 of the 16 boys are left-handed and 2 of the girls are left-handed. One student is chosen at random . Which of the following represents a conditional probability ? Ineed of help with the units Please help me!A boy on a skateboard travels the first 600m of a trip at an average speed of 2 m/s. He then travels the next 800m in 200s and spends the last 100s at a speed of 5 m/s. Find the average speed of the bicyclist for this trip. (x + 5x - 2)-(3x-x+4) 9. The graph is a plot of the velocity versus time for an objectmoving in a straight line. The x position of the objectat t = 0 seconds is 0 meters. At what time after t = 0 secondsdoes the object again pass through its initial position?(a) 1 second(b) Between 1 and 2 seconds(c) 2 seconds(d) 3 seconds Using only a compass and straightedge, construct an isosceles triangle with base and legs congruent to the segments shown below. Jason hangs sheetrock for a local contractor. His job requires him to hang the sheetrock overhead on ceilings. What type of ergonomic-related hazard is jason likely to encounter?. Enter an algebrak expression for the word expression. twice a number, minus 19 The expression is ? Giving a test to a group of students, the grades and gender are summarized below A B C TotalMale214 4 20Female1018 13 41Total1232 17 61If one student is chosen at random,Find the probability that the student did NOT get an "C". Round your answer to 3 decimal places_____. 3 batteries cost $5r and 8 folders cost $2r. Jason bought6 batteries and 4 folders. How much does he pay?Give your answer in terms of the (7 x 10^-5) x (5 * 10^-8)= ?x 10^ I need to write and simplify an algebraic expression for the perimeter of each shape.please help! Find the distance between vertices A and C of a regular hexagon whose sides are 20 cm each angle of the hexagon is 120 degrees In clinical applications, the unit parts per million (ppm) is used to express very small concentrations of solute, where 1 ppm is equivalent to 1 mg of solute per 1 L of solution. Calculate the concentration in parts per million for each of the solutions.There is 43 g of calcium in a total volume of 87 mL .concentration of calcium: _________ppm There is 0.91 mg of caffeine in a total volume of 105 mL .concentration of caffeine: _________ppm There is 0.47 mg of trace particles in a total volume of 95 dL .concentration of trace particles: ___________ What types of planning can be done to im prove a nations economy?A nation can undergo ____ planning or ___ planning in order to improve its economy. If $163,300 is invested in an account earning 3.75% annual interest compounded semi-annually, how much interest is accrued in the first 4 years? Round to the nearest cent? : Round your answer to the nearest hundredth. B 2 7 which is the better buy and provide the unit price for your answer! $5.28 for 6 candy bars or $12.75 for 15 candy bars? Use the graph to determine the point of intersection(4,5)(1,3)(5,4)(3,1)